High School Math

The following problems can be solved using only topics and theorems one encounters in the high school (Polynomials, Calculus, Probability/Statistics, Trigonometry, Analytic Geometry, Induction, etc).

  1. (Martin Gardner) There are n n racecars on a circular track. Amongst the nn cars, there is enough gas for one car to make a complete loop around the track. Show that there exists a car that can make a complete loop around the track by pooling gas from every car that it passes.

  2. (USAMO '73) Find all complex solutions to the following system of equations: x+y+z=3x2+y2+z2=3x3+y3+z3=3. \begin{aligned} x+y+z & = 3\\ x^2+y^2+z^2 & =3\\ x^3+y^3+z^3 & = 3.\\ \end{aligned}

  3. Find all real solutions to the following system of equations: x+y+z=3x2+y2+z2=3. \begin{aligned} x+y+z & = 3\\ x^2 + y^2+z^2 & =3. \\ \end{aligned}

  4. (Live Challenge Number Theory 5) Compute the last 3 digits of 171172 171^{172} .

  5. (Britain '87) f(x) f(x) is a polynomial with integer coefficients satisfying f(21)=17 f(21)=17, f(32)=247 f(32)=-247, and f(37)=33 f(37)=33. If there is an integer value N N such that f(N)=N+51 f(N)=N+51, show that N=26 N=26.

  6. (Live Challenge Algebra 4) Let x x be a real number such that 0.170<logx2<0.171 0.170 < \log_x 2 < 0.171 and 0.270<logx3<0.271 0.270 < \log_x 3 < 0.271. Determine the integer that is closest in value to logx7100 \log_x 7^{100} .

  7. (IMO'68) Determine all triangles whose side lengths are consecutive positive integers such that one angle is twice of another angle.

  8. (*) Calvin and Dan play a game of chance. A fair coin is flipped and the sequence of Heads and Tails is recorded. Calvin wins if the sequence HHH HHH appears first, and Dan wins if the sequence TTHH TTHH appears first. Who has a higher probability of winning?

Part 2 is available here.

#Algebra #KeyTechniques #Olympiad

Note by Calvin Lin
7 years, 2 months ago

No vote yet
1 vote

  Easy Math Editor

This discussion board is a place to discuss our Daily Challenges and the math and science related to those challenges. Explanations are more than just a solution — they should explain the steps and thinking strategies that you used to obtain the solution. Comments should further the discussion of math and science.

When posting on Brilliant:

  • Use the emojis to react to an explanation, whether you're congratulating a job well done , or just really confused .
  • Ask specific questions about the challenge or the steps in somebody's explanation. Well-posed questions can add a lot to the discussion, but posting "I don't understand!" doesn't help anyone.
  • Try to contribute something new to the discussion, whether it is an extension, generalization or other idea related to the challenge.
  • Stay on topic — we're all here to learn more about math and science, not to hear about your favorite get-rich-quick scheme or current world events.

MarkdownAppears as
*italics* or _italics_ italics
**bold** or __bold__ bold

- bulleted
- list

  • bulleted
  • list

1. numbered
2. list

  1. numbered
  2. list
Note: you must add a full line of space before and after lists for them to show up correctly
paragraph 1

paragraph 2

paragraph 1

paragraph 2

[example link](https://brilliant.org)example link
> This is a quote
This is a quote
    # I indented these lines
    # 4 spaces, and now they show
    # up as a code block.

    print "hello world"
# I indented these lines
# 4 spaces, and now they show
# up as a code block.

print "hello world"
MathAppears as
Remember to wrap math in \( ... \) or \[ ... \] to ensure proper formatting.
2 \times 3 2×3 2 \times 3
2^{34} 234 2^{34}
a_{i-1} ai1 a_{i-1}
\frac{2}{3} 23 \frac{2}{3}
\sqrt{2} 2 \sqrt{2}
\sum_{i=1}^3 i=13 \sum_{i=1}^3
\sin \theta sinθ \sin \theta
\boxed{123} 123 \boxed{123}

Comments

SOLUTION #2

{x+y+z=3x2+y2+z2=3x3+y3+z3=3\left\{\begin{array}{l}x+y+z=3\\ x^2+y^2+z^2=3\\ x^3+y^3+z^3=3\end{array}\right.

Consider the polynomial P(x)=x3+ax2+bx+cP(x)=x^3+ax^2+bx+c with roots p,q,rp,q,r. We use newton's sums to obtain that p+q+r=ap+q+r=-a p2+q2+r2=a22bp^2+q^2+r^2=a^2-2b p3+q3+r3=3aba33cp^3+q^3+r^3=3ab-a^3-3c

Now let the roots of this cubic be the variables in our equation. Thus, we have a=3-a=3 a22b=3a^2-2b=3 3aba33c=33ab-a^3-3c=3

From the first equation, we have a=3\boxed{a=-3}

Plugging into the second equation, we have b=3\boxed{b=3}

Plugging these into the third equation, we have c=1\boxed{c=-1}

Thus, our polynomial P(x)=x33x2+3x1P(x)=x^3-3x^2+3x-1 or P(x)=(x1)3P(x)=(x-1)^3

Since the only roots are p=q=r=1p=q=r=1, the only solution we have for our original system of equations is (x,y,z)=(1,1,1)\boxed{(x,y,z)=(1,1,1)}

Daniel Liu - 7 years ago

Log in to reply

Nice I did the same thing no need to repeat

Mardokay Mosazghi - 7 years ago

They did ask for all complex solutions and I have found (x,y,z)=(e2πi3,e4πi3,1) (x,y,z) = (e^\frac{2 \pi i}{3}, e^\frac{4 \pi i}{3}, 1) to be a solution as well by the formula for the roots of unity. By permuting x, y and z in this solution, we also get the following solutions: (e4πi3,e2πi3,1) (e^\frac{4 \pi i}{3}, e^\frac{2 \pi i}{3}, 1) , (1,e2πi3,e4πi3) (1, e^\frac{2 \pi i}{3}, e^\frac{4 \pi i}{3}) , (1,e4πi3,e2πi3) (1, e^\frac{4 \pi i}{3}, e^\frac{2 \pi i}{3}) , (e2πi3,1,e4πi3) (e^\frac{2 \pi i}{3}, 1, e^\frac{4 \pi i}{3}) and (e2πi3,1,e4πi3) (e^\frac{2 \pi i}{3}, 1, e^\frac{4 \pi i}{3}) , which, in addition to the trivial (1,1,1) (1,1,1) solution, leaves us with all complex solutions for the given system of equations.

Alaa Qarooni - 7 years ago

Log in to reply

Simple inspection of your answer of (e2πi3,e4πi3,1)\left(e^{\frac{2\pi i}{3}}, e^{\frac{4\pi i}{3}},1\right), we can determine that this and all of the various permutations are not solutions because they don't even satisfy x+y+z=3x+y+z=3. The only solution to problem 2 is (1,1,1) (1,1,1).

SOLUTION #1

Let us call the cars gas stations instead, to make the problem easier to understand.

First, let's suppose that the car has a large amount of gas initially. As it goes around the loop once, it begins and ends with the same amount of gas. In addition, there must be one gas station where the car arrives with the least amount of gas. But we're done: start at this gas station, and the car will never go lower than 00 gas, thus it can make it around the track once.

Daniel Liu - 7 years ago

Sorry no solution here, but I do have a question:

Is there any particular place where I can learn to solve problems like these?

Nathan Ramesh - 7 years ago

My attempt for SOLUTION #3

Didn't see a similar solution to mine, thought it was worth a share :)

We are given: a+b+c=3(1)a+b+c=3 \tag{1} a2+b2+c2=3(2)a^{2}+b^{2}+c^{2}=3 \tag{2}

From (1)(1), c=3abc=3-a-b.

Plugging that into (2)(2) we obtain, a2+b2+(3ab)2=3a^{2}+b^{2}+(3-a-b)^{2} = 3

Expanding the brackets and simplifying it we attain, a2+ab3a+b23b+3=0a^{2}+ab-3a+b^{2}-3b+3=0

It can be rewritten as, a2+a(b3)+(b23b+3)=0a^{2} + a(b-3) + (b^{2}-3b+3) = 0

Hence, using quadratic formula: a=(b3)±(b3)24(b23b+3)2(3)a = \frac{-(b-3) \pm \sqrt{(b-3)^{2}-4(b^{2}-3b+3)}}{2} \tag{3}

The discriminant must be 0 \ge 0 to have real roots so, (b3)24(b23b+3)0 (b-3)^{2}-4(b^{2}-3b+3) \ge 0

Simplifying the quadratic inequality above we obtain, 3(b1)20 -3(b-1)^{2} \ge 0

Solving the quadratic inequality we observe that the only solution for b is, b=1 \boxed{b=1} .

Hence, solving for a a using (3) (3) we obtain, a=1(13)±3(11)22=22 a = \frac{-1(1-3) \pm \sqrt{-3(1-1)^{2}}}{2} =\frac{2}{2}

a=1 \boxed{a=1}

Therefore, cc can be easily calculated using equation (1) (1), c=311=1 c = 3 - 1 -1 = 1

The only real solution to this question is:

(a,b,c)=(1,1,1) \boxed{ (a,b,c) = (1,1,1) }

Mohammad Al Ali - 7 years ago

Log in to reply

Nice. Is there an easier way to get at this?

Calvin Lin Staff - 7 years ago

Log in to reply

a+b+c=3a2+b2+c2=3whichimplies,a2+b2+c2abbcca=0=>12((a22ab+b2)+(a22ac+c2)+(b22bc+c2))=0=>(ab)2+(bc)2+(ac)2=0=>a=b=c=1a+b+c=3\\ { a }^{ 2 }+{ b }^{ 2 }+{ c }^{ 2 }=3\\ which\quad implies,{ a }^{ 2 }+{ b }^{ 2 }+{ c }^{ 2 }-ab-bc-ca=0\\ =>\frac { 1 }{ 2 } (({ a }^{ 2 }-2ab+{ b }^{ 2 })+({ a }^{ 2 }-2ac+{ c }^{ 2 })+({ b }^{ 2 }-2bc+{ c }^{ 2 }))=0\\ =>{ (a-b) }^{ 2 }+{ (b-c) }^{ 2 }+{ (a-c) }^{ 2 }=0\\ =>a=b=c=1

santhosh m - 7 years ago

Log in to reply

@Santhosh M Incredible! Nice solution. Clean, short and clear.

Mohammad Al Ali - 7 years ago

Log in to reply

@Mohammad Al Ali thanks!

santhosh m - 7 years ago

@Santhosh M that's what i approached good job buddy !!!

Rishabh Jain - 7 years ago

An easier way, as requested:

Let x+y+z=3x+y+z=3. We have that 3(x2+y2+z2)(x+y+z)23(x^2+y^2+z^2)\ge (x+y+z)^2 by Cauchy. Dividing by 33 on both sides, we have x2+y2+z2x+y+zx^2+y^2+z^2\ge x+y+z, where equality case is x=y=zx=y=z. However, since x2+y2+z2=x+y+zx^2+y^2+z^2=x+y+z, then we must have x=y=zx=y=z. Solving with this condition in mind easily gives us x=y=z=1x=y=z=1 as the only solution.

Daniel Liu - 7 years ago

Log in to reply

@Daniel Liu Thats a very nice way of solving it. The thing with me is i have very few knowledge of theorems heh..

Mohammad Al Ali - 7 years ago

Log in to reply

@Mohammad Al Ali AM-GM / Cauchy is a way to do it.

The other approach is to think of the geometric interpretation. We have a plane and a sphere, which intersect at the obvious point (1,1,1) (1, 1, 1) . To show that this is the unique point of intersection, it suffices to show that the plane is tangential to the sphere.

Calvin Lin Staff - 7 years ago

Log in to reply

@Calvin Lin Yes, I understand what you mean. But can anyone enlighten me on how do we prove a plane is tangential to a sphere? (What field of geometry is this? 3D??)

Happy Melodies - 7 years ago

Log in to reply

@Happy Melodies There is a variety of ways you can approach it.

If you are familiar with 3-d vectors, then you find the vector that is perpendicular to the plane, and show that is is parallel to the vector perpendicular to the sphere at the point of contact.

If you are familiar with multivariable calculus, you just apply the definition of tangency, which is a restatement of the above.

If you are familiar only with 2-d concepts, then take any plane and show that the intersection with this plane produces a circle and a line which are tangential.

Calvin Lin Staff - 7 years ago

Log in to reply

@Calvin Lin Oh.... I am not familiar with the above! :(

Happy Melodies - 7 years ago

@Daniel Liu How to rigorously prove that x2+y2+z2=x+y+z    x=y=z\displaystyle x^2+y^2+z^2=x+y+z\implies x=y=z?

mathh mathh - 6 years, 11 months ago

EDIT: real solutions. Oops. (ignore the rest)


Your conclusion is incorrect. One example of another solution is (0,12+32i,1232i)\left(0,-\dfrac{1}{2}+\dfrac{\sqrt{3}}{2}i,-\dfrac{1}{2}-\dfrac{\sqrt{3}}{2}i\right) and permutations.

However, proving that (1,1,1)(1,1,1) is the only real solution is an easy task. From my solution, this is when c+13\sqrt[3]{c+1} has only a real value which is only when c+1=0c+1=0 or c=1c=-1; then, the solution follows to be (1,1,1)(1,1,1).

Daniel Liu - 7 years ago

SOLUTION #4

Let's define, for convenience, a function f(x)f(x) that shows the last three digits of 171x171^x. We have, by grinding, f(3)=211,f(9)=931,f(27)=491,f(54)=81,f(162)=441,f(171)=571f(3)=211, f(9)=931, f(27)=491, f(54)=81, f(162)=441, f(171)=571 and finally f(172)=641.f(172)=\boxed{641.}

Guilherme Dela Corte - 7 years ago

Log in to reply

sorry, I cannot understand this grinding.

Kartik Sharma - 7 years ago

Log in to reply

Explanation:

1713=5000211171^3 = 5000211, so the last three digits of 1713171^3 are 211211 (f(3)=211f(3)=211). 1719=(1713)3=125015825667824393931171^9 = (171^3)^3 = 125015825667824393931, which has the same last three digits as 2113=9393931211^3 = 9393931, so f(9)=931f(9)=931. ...etc...

Guilherme Dela Corte - 7 years ago

Log in to reply

@Guilherme Dela Corte okay! got it, well thanks!

Kartik Sharma - 7 years ago

@Guilherme Dela Corte in the second step can we only do cubing/sq. of the last digits??

Ashwin Upadhyay - 6 years, 8 months ago

solution 7

Let the sides be a,a+1,a+2 Let the angle opposite side to a be A,opposite to b be a+1,and side opposite to c be a+2 By using cosine rule, cosA=a+5/2a+4,cosB=a+1/2a and cosC=a-3/2a Using cos2x=2cos^2X-1 We get a=4 for A=2C a=1 for A=2B But a cannot be 1 as difference of two sides should be less than the third side Therefore sides are 4,5 and 6

Prateek Bontha - 7 years ago

SOLUTION #3

Similarly to solution #2, we make the polynomial P(x)P(x) and obtain a=3a=-3 and b=3b=3. However, we cannot tell what cc is.

Thus, our polynomial is P(x)=x33x2+3x+cP(x)=x^3-3x^2+3x+c or P(x)=(x1)3+c+1P(x)=(x-1)^3+c+1

Solving for xx, we obtain x1=c+13x-1=-\sqrt[3]{c+1} or x=1c+13x=1-\sqrt[3]{c+1}

Note that there are 3 distinct values of c+13\sqrt[3]{c+1}. These three values correspond to the three solutions.

Daniel Liu - 7 years ago

Log in to reply

The question asks for real solutions.

Calvin Lin Staff - 7 years ago

Log in to reply

Aww darn oops. But I can still generate the real solution by letting c=1c=-1.

Daniel Liu - 7 years ago
×

Problem Loading...

Note Loading...

Set Loading...